Wyznacznik macierzy metoda Chió gdzie mam błąd?

Przestrzenie wektorowe, bazy, liniowa niezależność, macierze.... Formy kwadratowe, twierdzenia o klasyfikacji...
Christofanow
Użytkownik
Użytkownik
Posty: 174
Rejestracja: 30 sie 2010, o 12:39
Płeć: Mężczyzna
Lokalizacja: ffff
Podziękował: 12 razy
Pomógł: 2 razy

Wyznacznik macierzy metoda Chió gdzie mam błąd?

Post autor: Christofanow »

Wyznacznik:
\(\displaystyle{ \begin{vmatrix} 1&1&-1\\-2&-3&4\\1&0&-3 \\\end{vmatrix} =
\begin{vmatrix}
\begin{vmatrix}1&1 \\ -2& -3\end{vmatrix} & \begin{vmatrix}1&-1 \\ -2&4\end{vmatrix}\\
\begin{vmatrix}1&1 \\1&0\end{vmatrix}& \begin{vmatrix}1&-1 \\1&-3\end{vmatrix}
\end{vmatrix} = \begin{vmatrix}-1&2 \\ -1& -4\end{vmatrix} = 6}\)
a poprawna wartość wyznacznika to 4. Gdzie zrobiłem błąd?
bartek118
Użytkownik
Użytkownik
Posty: 5974
Rejestracja: 28 lut 2010, o 19:45
Płeć: Mężczyzna
Lokalizacja: Toruń
Podziękował: 15 razy
Pomógł: 1251 razy

Wyznacznik macierzy metoda Chió gdzie mam błąd?

Post autor: bartek118 »

\(\displaystyle{ \begin{vmatrix} 1&1&-1\\-2&-3&4\\1&0&-3 \\\end{vmatrix} = \begin{vmatrix} \begin{vmatrix}1&1 \\ -2& -3\end{vmatrix} & \begin{vmatrix}1&-1 \\ -2&4\end{vmatrix}\\ \begin{vmatrix}1&1 \\1&0\end{vmatrix}& \begin{vmatrix}1&-1 \\1&-3\end{vmatrix} \end{vmatrix} = \begin{vmatrix}-1&2 \\ -1& -2\end{vmatrix} = 4}\)
Christofanow
Użytkownik
Użytkownik
Posty: 174
Rejestracja: 30 sie 2010, o 12:39
Płeć: Mężczyzna
Lokalizacja: ffff
Podziękował: 12 razy
Pomógł: 2 razy

Wyznacznik macierzy metoda Chió gdzie mam błąd?

Post autor: Christofanow »

O! Dziękuję za wskazanie błędu.

-- 27 lip 2013, o 21:00 --

\(\displaystyle{ \begin{vmatrix} 12&6&-4\\6&4&4\\3&2&8 \\\end{vmatrix} =
\frac{1}{12}
\begin{vmatrix}
\begin{vmatrix}12&6 \\ 6& 4 \end{vmatrix} & \begin{vmatrix}12&-4 \\ 6&4 \end{vmatrix} \\
\begin{vmatrix}12 & 6 \\ 3 &2 \end{vmatrix} & \begin{vmatrix}12&6 \\ 6& 4 \end{vmatrix}
\end{vmatrix}
= \frac{1}{12}
\begin{vmatrix}
12&72 \\ 6&108
\end{vmatrix}
= \frac{1}{12} * 864 = 72}\)

Wyznacznik się zgadza. Pytanie moje jest następujące - jak interpretować \(\displaystyle{ \frac{1}{12}
\begin{vmatrix}
12&72 \\ 6&108
\end{vmatrix}}\)
Czy mogę każdy element wyznacznika przemnożyć przez ułamek? Wówczas otrzymuję wyznacznik \(\displaystyle{ \begin{vmatrix}
1&6 \\ \frac{1}{2} & 9 = 6
\end{vmatrix}}\)
O co tutaj chodzi? Czyli mnożenie przez ułamek można zastosować dopiero na wartości liczbowej wyznacznika a nie na jego składnikach?

-- 27 lip 2013, o 21:01 --

Edit: wino być:
\(\displaystyle{ \begin{vmatrix} 1&6 \\ \frac{1}{2} & 9 \end{vmatrix} = = 6}\)-- 27 lip 2013, o 21:02 --Dlaczego nie mogę edytować wypowiedzi?
ODPOWIEDZ